Double slit experiment - QM or wave?

In summary, the double slit experiment is a fundamental experiment in quantum mechanics that demonstrates the wave-particle duality of matter. It shows that particles, such as electrons, can exhibit both wave-like and particle-like behaviors, depending on how they are observed. This experiment has played a crucial role in shaping our understanding of quantum mechanics and the nature of reality. Despite ongoing debates, the consensus is that the double slit experiment supports the wave theory of quantum mechanics, rather than the particle theory.
  • #1
Hydr0matic
197
1
I propose a setup to distinguish between the quantum interpretation and the wave interpretation of the double slit experiment.

Briefly, they are:

Quantum
The photon gun fires a photon that travels through either slit A or slit B, then hits a detector cell registering a spot in the detector pattern. All photons follow the same procedure, and with the help of quantum mechanics, the result is a nice interference pattern.

Wave
The photon gun fires a photon that travels through both slit A and slit B, then hits a detector cell with either destructive or constructive interference, depending on the location of the detector cell. If the superposition is "too destructive", no spot will register in the detector pattern. All photons interfere destructively or constructively and the result is a nice interference pattern.


So how do we distinguish between these interpretations? We do it by counting detections.

If the photon gun fires 1000 photons per minute, quantum mechanics predict that, during a minute, a 1000 detections will be registered (ideally). The wave theory on the other hand, predict that during a minute, only photons that are not interfering destructively will register at the detector.

So the setup is simple. First fire the photon gun without the double slit to determine the photon firing rate, then add the double slit and count the detections.

Since I adhere to the wave interpretation, I predict that there will be a significant difference between firing rate and detection rate. However, if this was the case, you could still argue that the non-detections "got lost" somewhere at the slits. But even if this was the case, you could still distinguish the theories from each other. Because, if the quantum interpretation is correct, the "lost photons" are lost, just random mishaps at the slits. On the other hand, in the wave interpretation all "lost photons" are destructive interference. This means that, if we alter the detector so that the destructive areas (dark bands) become constructive instead of destructive, there should be a significant increase in detections which cannot be explained by quantum mechanics.

An illustration of the setup:
http://physics.soderholms.com/doubleslit_setup.jpg


Any thoughts?
 
Last edited by a moderator:
Physics news on Phys.org
  • #2
Hate to break this to you, but there's a more elegant experimental demonstration that has been done for undergraduate physics labs based on the Mach-Zehnder interferometer. See T.L. Dimitrova and A. Weis, Am. J. Phys. v.76, p.137 (2008).

One of the component of the experiment that was done by the students was this:

The demonstration, whose result is astonishing for students, is realized in the following way. First the fringe pattern is locked to a photodiode as explained in Sec. IV B, and the photomultiplier is moved to a fringe minimum, as characterized by a low photon count rate which can also be displayed acoustically. If now path A of beam 1 is blocked inside the interferometer, it is possible to hear (and see) a distinct increase of the click rate. This result demonstrates that if we give each photon the choice of taking either path A or path B, it has a low probability to appear at the detector. In contrast, if we force the photon to follow a specific path by blocking the other path, then the probability to arrive at the detector is much higher. The puzzling fact that a two-path alternative for each photon prevents it from reaching the detector, while blocking one of the paths leads to a revival of the clicks, is most intriguing for beginning students. This experiment is well suited for illustrating this remarkable quantum mechanical effect, which can be explained only if we assume that each photon simultaneously takes both paths A and B; that is, each photon, in the phrasing of Dirac, "interferes with itself."

You'll have a tough time convincing anyone that your "wave interpretation" is valid with such observation.

Zz.
 
  • #3
ZapperZ said:
You'll have a tough time convincing anyone that your "wave interpretation" is valid with such observation.
Oook? .. The quote you supplied completely proves my point. I don't get it.

The "revival of the clicks" cannot be interpreted as anything other than elimination of destructive interference.

I'm guessing this is more of a semantics issue now then. What would you say is the difference between a wave and a photon that "interferes with itself"?
 
  • #4
Hydr0matic said:
Oook? .. The quote you supplied completely proves my point. I don't get it.

The "revival of the clicks" cannot be interpreted as anything other than elimination of destructive interference.

I'm guessing this is more of a semantics issue now then. What would you say is the difference between a wave and a photon that "interferes with itself"?

The difference happened at the beam splitter. You need to read the entire article, and also the http://people.whitman.edu/~beckmk/QM/grangier/Thorn_ajp.pdf" .

Zz.
 
Last edited by a moderator:
  • #5
ZapperZ said:
The difference happened at the beam splitter. You need to read the entire article, and also the http://people.whitman.edu/~beckmk/QM/grangier/Thorn_ajp.pdf" .

Zz.
I don't have access to the article via AJP. The Thorn paper seems to be more of a bell test setup with beam splitting and coincidence counts, i.e. no interference. So how is it relevant?

Concerning duality, different experiments show different things, and one does not take precedence over another (even if the wave interpretation is more easily observed). My original point was that the double slit is one of those experiments that cannot be explained satisfactory with a quanta of light. But you obviously believe otherwise, so I ask you. Why is the double slit not explained with a wave model of light? Which interpretation would Occam prefer?
 
Last edited by a moderator:
  • #6
Hydr0matic said:
I don't have access to the article via AJP. The Thorn paper seems to be more of a bell test setup with beam splitting and coincidence counts, i.e. no interference. So how is it relevant?

The point being that "interference" by itself isn't sufficient, because both scenario results in the same observation (unless you want to consider higher order interference). What is important is the "which-way" observation that can't be resolved via the wave scenario.

Concerning duality, different experiments show different things, and one does not take precedence over another (even if the wave interpretation is more easily observed). My original point was that the double slit is one of those experiments that cannot be explained satisfactory with a quanta of light. But you obviously believe otherwise, so I ask you. Why is the double slit not explained with a wave model of light? Which interpretation would Occam prefer?

But it can't. If you look at ALL the observation, your application of Occam will require the minimal sent of description to describe ALL, not just one, not just two, observations. I haven't even yet invoked other phenomena such as the antibunching observation, multiphoton photoemission, etc. that current has zero description via wave scenario. Do you think your Occam principle will allow you to switch gears of describing light one way for one set of experiments, and switching to another to describe another set? I can stick to just one for all of them.

Zz.
 
  • #7
Hydr0matic said:
If the photon gun fires 1000 photons per minute, quantum mechanics predict that, during a minute, a 1000 detections will be registered (ideally). The wave theory on the other hand, predict that during a minute, only photons that are not interfering destructively will register at the detector.

So the setup is simple. First fire the photon gun without the double slit to determine the photon firing rate, then add the double slit and count the detections.

Since I adhere to the wave interpretation, I predict that there will be a significant difference between firing rate and detection rate.

The whole idea is completely flawed. Interference is a phenomenon arising due to superposition of possible paths/probability amplitudes. There is no such thing as photons just interfering destructively and vanishing completely. For every zone of destructive interference, there is also a zone of constructive interference, so the total photon number does not change anyway. Otherwise this would be a harsh violation of energy conservation.
 
  • #8
ZapperZ said:
The point being that "interference" by itself isn't sufficient, because both scenario results in the same observation (unless you want to consider higher order interference). What is important is the "which-way" observation that can't be resolved via the wave scenario.
Which scenarios? and what about the "which-way" observation can't be resolved?

Both my setup, the quote you supplied, and Cthugha's post tells you why interference is enough, and both interpretations do not result in equal observations. QM predicts no change in clicks / number of detections when blocking one path or changing the resulting interference pattern. Wave theory does.


ZapperZ said:
But it can't. If you look at ALL the observation, your application of Occam will require the minimal sent of description to describe ALL, not just one, not just two, observations. I haven't even yet invoked other phenomena such as the antibunching observation, multiphoton photoemission, etc. that current has zero description via wave scenario. Do you think your Occam principle will allow you to switch gears of describing light one way for one set of experiments, and switching to another to describe another set? I can stick to just one for all of them.
I'm not sure you can. How do you fit longer wavelength radiation in your description? radio transmission, microwave ovens... and refraction, diffraction? .. Even if you could, would you consider your description to be the final say in the matter? I can certainly appreciate the benefits and practicality of having a single description for all phenomena, but that doesn't mean I'm ruling out all other possibilities.

So please, I'm interested... Just looking at the double slit experiment, what about it makes you consider it to be better explained with quantum mechanics than with a wave model? Because I've always wondered why physicists use it as an example of how special QM is, when the actual explanation involves quanta of light "interfering with itself". What does this mean, and how does it differ from a wave model?


Cthugha said:
The whole idea is completely flawed. Interference is a phenomenon arising due to superposition of possible paths/probability amplitudes. There is no such thing as photons just interfering destructively and vanishing completely. For every zone of destructive interference, there is also a zone of constructive interference, so the total photon number does not change anyway. Otherwise this would be a harsh violation of energy conservation.
Read the quote in ZapperZ first post. Your claim is not true locally in an experiment like this, the photon detection does change. At the position where there is destructive interference, the light doesn't interact with the matter (eg. the detector), so it continues it's propagation and eventually interferes differently and interacts with something else. Energy is conserved, but not confined to the experiment.
 
  • #9
Hydr0matic said:
QM predicts no change in clicks / number of detections when blocking one path or changing the resulting interference pattern. Wave theory does.
You have grossly miss read something somewhere. Half the clicks with no interferance pattern just a dispersion patttern just like wave theroy does is excactly what QM says.

Just google for double slit basics or reread whatever your looking at.
 
  • #10
Hydr0matic said:
Which scenarios? and what about the "which-way" observation can't be resolved?

In a beam splitter with wave picture, half of the way goes through while the other half gets reflected. In a photon picture, a photon has a probability of 50% to go through and 50% to get reflected. This means that if you put your detector over both branch of the interferometer, you'll only get one detection, not both, in the photon picture. This is what the J.J. Thorn et al. expt. detected.

Both my setup, the quote you supplied, and Cthugha's post tells you why interference is enough, and both interpretations do not result in equal observations. QM predicts no change in clicks / number of detections when blocking one path or changing the resulting interference pattern. Wave theory does.

I don't see how Cthugha's post contradicts mine. I described a generic interference experiment and that you can described it with both pictures.

I'm not sure you can. How do you fit longer wavelength radiation in your description? radio transmission, microwave ovens... and refraction, diffraction? .. Even if you could, would you consider your description to be the final say in the matter? I can certainly appreciate the benefits and practicality of having a single description for all phenomena, but that doesn't mean I'm ruling out all other possibilities.

Actually, yes you can since YOU were the one who wanted to invoke Occam. Refraction and diffraction requires interaction with the material, and in condensed matter physics, optical transport are often described using the photon picture. See for example, the microscopic explanation on the interaction of photons with the phonon structure of a material that can result in the explanation for why things are opaque, transparent, and why light moves "slower" in a dispersive material.

So please, I'm interested... Just looking at the double slit experiment, what about it makes you consider it to be better explained with quantum mechanics than with a wave model? Because I've always wondered why physicists use it as an example of how special QM is, when the actual explanation involves quanta of light "interfering with itself". What does this mean, and how does it differ from a wave model?

Shoot one photon at a time, i.e. light with extremely low intensity. Look at it over time. Now do the same with electrons, protons, etc. Tell me if what you observe is different. If yes, then light is different than the "physical particles" such as electron, protons, etc. and you may have a leg to stand on. If they look the same and have the same characteristics, then you have a lot of explaining to do.

Zz.
 
  • #11
Hydr0matic said:
Read the quote in ZapperZ first post. Your claim is not true locally in an experiment like this, the photon detection does change. At the position where there is destructive interference, the light doesn't interact with the matter (eg. the detector), so it continues it's propagation and eventually interferes differently and interacts with something else. Energy is conserved, but not confined to the experiment.

Of course the photon detection rate is changed locally. This is what interference is about. So the total count rate of the detector will only change, if the detector is small compared to the total dimensions of your interference pattern. But if you use a small detector and place it inside a spot of destructive interference, qm also tells you, there will be a significant difference between the counting rate with and without double slit in this spot. So I do not see any difference.

The only possibility to have no change in the counting rates would be to use a deterministic photon gun, which fires single photons in completely defined directions at completely defined moments. This would not produce an interference pattern. However this case corresponds to the case of completely incoherent light, where the distance of the slits is far longer than the coherence length of your wave. So any wave model will not predict any changes in the counting rates either.
 
  • #12
Cthugha beat me to it. I'm afraid it looks like your conceptual understanding of the two-slit experiment is wrong. The experiment has been performed many times before, and the QM description wins.
Suppose your "wave" interpretation
Hydr0matic said:
The photon gun fires a photon that travels through both slit A and slit B, then hits a detector cell with either destructive or constructive interference, depending on the location of the detector cell. If the superposition is "too destructive", no spot will register in the detector pattern. All photons interfere destructively or constructively and the result is a nice interference pattern.
were correct. Then a single photon, traveling through the slits, would produce dots at the location of every constructive interference fringe. This simply isn't what you see when you do the experiment.
Also, it's technically wrong to say that in the quantum description
The photon gun fires a photon that travels through either slit A or slit B, then hits a detector cell registering a spot in the detector pattern.
The WHOLE POINT of QM is that this is NOT what we say happens. What we say happens in QM is that there is a wavefunction, which goes through both slits, and interferes with itself constructively and destructively at the interferences fringes along the screen. The square of the amplitude of this wavefuction at a point describes the probability that an electron will be detected at that point if we choose to make a measurement there. So the interference pattern is slowly manifested on the screen as increasing numbers of photons are detected in the places earmarked as 'probable' by the wavefunction. When a photon is detected in one place, its wavefunction is said to "collapse", and there's no longer any chance of finding it anywhere else.
 
Last edited:
  • #13
Hydr0matic said:
So please, I'm interested... Just looking at the double slit experiment, what about it makes you consider it to be better explained with quantum mechanics than with a wave model? Because I've always wondered why physicists use it as an example of how special QM is, when the actual explanation involves quanta of light "interfering with itself". What does this mean, and how does it differ from a wave model?

I think you're right about the double slit experiment. Feynman once said that all the essential mysteries of QM are included in the double slit experiment, but he was talking about
a thought experiment using electrons, not light. He meant that the essential mystery is that particles (like electrons) show interference. He never said anything about the double slit experiment with regard to the wave vs particle theories of light.

People also invoke the image being built up one click at a time, or one silver halide crystal at a time as evidence of the particle nature of light. This argument of course has nothing to do with double slits or triple slits or any kind of interference. It seems convincing at first glance but is actually quite hard to nail down. The problem is that it's hard to know when you have exactly one photon at a time: and both the wave theory and the particle theory allow for a diminishing probability of coincidence counts as you make the source weaker and weaker.
 
  • #14
The evidence that light has some particulate nature is nothing to do with the two-slit experiment. It's to do with the photoelectric effect.
 
  • #15
muppet said:
The evidence that light has some particulate nature is nothing to do with the two-slit experiment. It's to do with the photoelectric effect.
I'm not disagreeing with you, just would like to add that, however, the two slits experiment, in my hopinion, has to do with the fact that a corpuscle of light has or not a definite individuality from the source to the detectors. What I mean is this:

If a corpuscle with such individuality exists, it must go through one or the other or both (splitting itself) the slits. It's well known that if it goes through one or the other, you don't get an interference pattern; it's probably less known that if it splits in two and goes through both...nothing changes because you have two particles passing through the slits and the same you don't have an interference pattern (ah yes, you could say it interferes with itself, but this is what a wave does, not a corpuscle).

The only possibility, in my opinion, is that the photon has not a definite individuality between the source and the detectors, the field constantly creates and destroyes it, so it's the field that passes through both slits (exactly how you wrote), interfering with itself and then creating the single photon which hits the detectors.

Just another crazy idea, of course. :smile:
 
Last edited:
  • #16
Hey all. I appreciate you joining the discussion, but I really would like to narrow this thread down. Wave-particle duality is a huge subject that can be debated forever, so I would appreciate if we could focus on the original statement. Which was...

• QM and wave theory predicts different outcomes in the double slit experiment. Wave theory predicts a significant decrease in detection rate compared to firing rate, QM does not.

Is there any data on this subject?

Cthugha said:
Of course the photon detection rate is changed locally. This is what interference is about. So the total count rate of the detector will only change, if the detector is small compared to the total dimensions of your interference pattern. But if you use a small detector and place it inside a spot of destructive interference, qm also tells you, there will be a significant difference between the counting rate with and without double slit in this spot. So I do not see any difference.
What I'm talking about, is using a detector that will cover the entire area behind the slits, so that all photons passing the slits can be detected. In that case, QM predicts that... when a photon is fired, it's probabillity wave will pass the slits, interfere, and collapse at the detector determining where the photon will strike. This means that all (ideally) photons will strike the detector. This is NOT what wave theory predicts. In wave theory, the dark fringes in the pattern is a result of the fact that this area was hit by a photon (wave) that was interfering destructively, and therefore didn't set off the detector. This means that wave theory predicts a significant decrease in detection rate compared to firing rate. Unlike QM.



Off Topic:

ZapperZ said:
See for example, the microscopic explanation on the interaction of photons with the phonon structure of a material that can result in the explanation for why things are opaque, transparent, and why light moves "slower" in a dispersive material.
And why would a quanta bend it's path when interacting with the material?


ZapperZ said:
Shoot one photon at a time, i.e. light with extremely low intensity. Look at it over time. Now do the same with electrons, protons, etc. Tell me if what you observe is different. If yes, then light is different than the "physical particles" such as electron, protons, etc. and you may have a leg to stand on. If they look the same and have the same characteristics, then you have a lot of explaining to do.
You know as well as me that these experiments are on a completely different scale when shooting particles. Based on what I've been able to find on the web the setup is very different, but results are kind of similar, yes. They both display fringes anyway. I have some questions though. Have a look at the video on this site: http://www.hqrd.hitachi.co.jp/em/doubleslit.cfm
Here's a quick analysis of the brightness of the fringes: http://physics.soderholms.com/electron_fringes.jpg
Which fringe would you say is the center one? A typical double slit pattern has an intensity distribution looking something like this: http://img.sparknotes.com/figures/C/c33e2bffc162212e1d9aa769ad3ae54f/envelope.gif
It looks to me like, either there's something shifty going on with the intensities, or the center fringe is a minimum. So I'm not convinced it's actually an interference pattern we're looking at. In the setup (http://physics.soderholms.com/electron_biprism.jpg ) there's a few (to say the least) things in the way of the detector. And the paper (American Journal of Physics 57 117-120) doesn't say anything about what it looks like without the biprism, or what happens if you change the distance to the detector, or block one of the paths. It's basically an experiment set up and performed only to confirm quantum mechanics. They might as well just put the conclusion in there.

muppet said:
Suppose your "wave" interpretation were correct. Then a single photon, traveling through the slits, would produce dots at the location of every constructive interference fringe. This simply isn't what you see when you do the experiment.
What are you saying, that light can't be wave because we can shine a dot on the wall? That light must be corpuscular because it travels in straight lines? That's a bit 18th century of you.

Anyway. Too much off topic now.
 
Last edited by a moderator:
  • #17
Hydr0matic said:
Wave-particle duality is a huge subject that can be debated forever, so I would appreciate if we could focus on the original statement. Which was...

• QM and wave theory predicts different outcomes in the double slit experiment. Wave theory predicts a significant decrease in detection rate compared to firing rate, QM does not.

Is there any data on this subject?

ZapperZ is correct and you are closing your eyes to an experiment that answers directly the question you are asking: whether photons are discrete or not. The Thorn experiment (which is simply an undergrad version of experiments previously performed by others) asks: are photons simply waves of probability? The answer is a big NO. There are definite predictive differences - as you imagine - between various possibilities. The one you suggest is clearly ruled out, by 377 standard deviations if I recall. The QM predictions are confirmed in the process. *Read* the paper. I might suggest you quit insisting that the experiment is not relevant when it obviously is. How can anyone take your suggestion seriously when you ignore facts you don't like?
 
  • #18
Hydr0matic said:
• QM and wave theory predicts different outcomes in the double slit experiment. Wave theory predicts a significant decrease in detection rate compared to firing rate, QM does not.
Sorry but I don't understand it. Can you elaborate? Also, where exactly is the difference between firing rate and detection rate?
What I'm talking about, is using a detector that will cover the entire area behind the slits, so that all photons passing the slits can be detected. In that case, QM predicts that... when a photon is fired, it's probabillity wave will pass the slits, interfere, and collapse at the detector determining where the photon will strike. This means that all (ideally) photons will strike the detector. This is NOT what wave theory predicts. In wave theory, the dark fringes in the pattern is a result of the fact that this area was hit by a photon (wave) that was interfering destructively, and therefore didn't set off the detector. This means that wave theory predicts a significant decrease in detection rate compared to firing rate. Unlike QM.
If 10 photons are sent in a time interval of one second, ten photons will be detected in a time interval of one second, wave theory or QM.

Also, when you say that:
Quantum
The photon gun fires a photon that travels through either slit A or slit B, then hits a detector cell registering a spot in the detector pattern. All photons follow the same procedure, and with the help of quantum mechanics, the result is a nice interference pattern.

Wave
The photon gun fires a photon that travels through both slit A and slit B, then hits a detector cell with either destructive or constructive interference, depending on the location of the detector cell. If the superposition is "too destructive", no spot will register in the detector pattern. All photons interfere destructively or constructively and the result is a nice interference pattern
1. Your description of the "Quantum interpretation" is wrong, as others have said, because "a photon that travels through either slit A or slit B" is exactly a photon of which you have the "which way" information, and that means no interference pattern. Example: you detect the photon near one of the slits putting there a little detector; clearly this is not the two slits experiment we are talking about.

2. About your wave interpretation. As I've written in my previos post, if you are certain that the particle splits in two and passes through both slits, nothing changes, it's as if you had two photons passing simultaneously through both slits; it could seem strange, but you have the "which way" information the same, and so no interference pattern. Think to the fact that to say "the particles splits in two and both passes through the slits" means that you have that specific information, and to have it you should put two little detectors near every slit and detect the two particles (the two "half photon") near the slits; so, no interference pattern. Unless you invoke De Broglie Bohm interpretation, the only possibility is that no particles passes through one or the other or both slits. (In my opinion).
 
  • #19
Hydr0matic said:
What are you saying, that light can't be wave because we can shine a dot on the wall? That light must be corpuscular because it travels in straight lines? That's a bit 18th century of you.
I'm not saying that light can't be a wave because we can shine a dot on the wall, and I'm certainly not saying that it's corpuscular because it travels in straight lines.
What I'm saying is that if light were a classical wave, then an arbitrarily small "amount" of light would show evidence of an interference pattern when diffracted through one or more slits. It's perfectly possible to "shine a dot on the wall" with a laser pointer (say), but if you take a laser pointer (emitting innumerable photons) and shine it through a diffraction grating you see an interference pattern, clear as day. If light were a pure, classical wave, then it wouldn't be possible to shine any amount of light through a two-slit setup in such a way that you didn't see interference fringes immediately.

Lightarrow: Schroedinger originally suggested that there were no particles, only localised wave packets. Lorentz (I think!) demonstrated that such "matter waves" would spread out over time due to dispersion, and lose their localisation, which of course conflicts with the existence extremely well localised particles in the world today. This is the reason why people don't think of the wavefunction as describing a particle as being physically spread out. I suppose I can't honestly say, however, that your idea is any crazier than the idea that a particle pops into existence when we decide to look for it :rolleyes:
 
  • #20
DrChinese said:
ZapperZ is correct and you are closing your eyes to an experiment that answers directly the question you are asking: whether photons are discrete or not.

That's not the question he was asking. Hydromatic is wrong on certain points of physics but he was quite clear on this point: he asks why people choose to use the double-slit experiment to support the particle theory over the wave theory. I wouldn't normally butt in on one of ZapperZ's discussions because of his tendency to use his power as a moderator to silence his opponents, but since you found it necessary to weigh in on his behalf, I thought it was only fair for me to point out that ZapperZ has not answered this question. In fact, the extensive quote he provides in his first answer shows that the wave theory rather than the particle theory is the obvious explanation for the double slit experiment. Hydromatic rightly pointed this out in his response.
 
  • #21
muppet said:
Lightarrow: Schroedinger originally suggested that there were no particles, only localised wave packets. Lorentz (I think!) demonstrated that such "matter waves" would spread out over time due to dispersion, and lose their localisation, which of course conflicts with the existence extremely well localised particles in the world today. This is the reason why people don't think of the wavefunction as describing a particle as being physically spread out. I suppose I can't honestly say, however, that your idea is any crazier than the idea that a particle pops into existence when we decide to look for it :rolleyes:
Ok, however I haven't actually said that there are no particles. What I say, I think, it's a little bit complex (but I can be totally wrong): the particles are present but they don't have a precise individuality, in the sense that we cannot say: the photon N. 109 has started from the source, now it's near the first slit, it has passed the first slit, now it has hit the detector N. 13,115. What I'm asserting, as personal idea (I'm sorry, sometimes I can't help having personal ideas, it's too appealing for me!) is that the photons are continuously created and destroyed by the field during the journey between source and detectors, so that it's not possible to say that a specific photon has passed through a specific slit and it's exactly for this reason, in my opinion, that we cannot have the "which way information" and have the interference pattern. Remember that the number of the photons, even if doesn't vary on average, it's not well determined: there is for example an indeterminacy relation between the number of photons and the phase of the electromagnetic field. The more one is determined, the less is the other, and you know that in an interference experiment the phase must be quite well defined (or you don't see it).
 
  • #22
Marty said:
That's not the question he was asking. Hydromatic is wrong on certain points of physics but he was quite clear on this point: he asks why people choose to use the double-slit experiment to support the particle theory over the wave theory. I wouldn't normally butt in on one of ZapperZ's discussions because of his tendency to use his power as a moderator to silence his opponents, but since you found it necessary to weigh in on his behalf, I thought it was only fair for me to point out that ZapperZ has not answered this question. In fact, the extensive quote he provides in his first answer shows that the wave theory rather than the particle theory is the obvious explanation for the double slit experiment. Hydromatic rightly pointed this out in his response.

Actually, I thought I HAD answered the question completely, and that I've stopped responding because I would simply be saying the SAME thing. If someone simply didn't care to accept what I've cited, then how many times does one have to say the same thing? And note that if I want to use my power as a moderator to silence my opponents, why didn't I shut down this thread already? The fact that it continued to go on clearly falsified your claim.

There are two separate issues here:

1. The OP's scenario is wrong in the first place. As has been mentioned many times by several people, the QM scenario of what would happen is simply not correct.

2. A double slit experiment can distinguish between "wave" and "particle" picture. Again, there's a huge amount of confusion here because the TYPE of such experiment. The Mach-Zenhder experiment is a perfectly valid "2-slit" experiment and allows for even more testing being done. And when this is done for low-intensity sources AND incorporating a which-way criteria, only then can one actually distinguish one from the other. again, refer to the J.J. Thorn et al. paper. In fact this is an experiment that can actually be done at the undergraduate level, so it isn't that difficult to do! One can easily test the OP's scenario using such set up!

So who's rushing to do this and get nominated for a Nobel Prize?

Zz.
 
  • #23
ZapperZ said:
There are two separate issues here:

1. The OP's scenario is wrong in the first place. As has been mentioned many times by several people, the QM scenario of what would happen is simply not correct.

2. A double slit experiment can distinguish between "wave" and "particle" picture. Again, there's a huge amount of confusion here because the TYPE of such experiment. The Mach-Zenhder experiment is a perfectly valid "2-slit" experiment and allows for even more testing being done. And when this is done for low-intensity sources AND incorporating a which-way criteria, only then can one actually distinguish one from the other. again, refer to the J.J. Thorn et al. paper. In fact this is an experiment that can actually be done at the undergraduate level, so it isn't that difficult to do! One can easily test the OP's scenario using such set up!
2. I'm not satisfied with a reference to this experiment because it has other components that we don't share the same interpretation about. So in order for me to argue the detection rate issue, I would first have to argue the beam splitting issue for example. And I would prefer not to because it's not really relevant to the double slit experiment, and the key issue is much simpler. All you need is a photon gun, a double slit, a detector and two values - one count rate when there is no double slit, and one when there is.

1. So, if I'm wrong, what would QM predict with regards to the detection rate vs firing rate?
 
  • #24
lightarrow said:
Sorry but I don't understand it. Can you elaborate? Also, where exactly is the difference between firing rate and detection rate?
Firing rate is the actual number of photons fired. The number you would detect if there is nothing in between the photon gun and the detector. The detection rate is the detectors count.

lightarrow said:
If 10 photons are sent in a time interval of one second, ten photons will be detected in a time interval of one second, wave theory or QM.
Not true. This is what QM predicts, not wave theory. In wave theory, there are just as many photons hitting the dark fringes as there is hitting the bright fringes, only they're not setting off the detector because in these exact positions, they are interering destructively. Therefore, the detection count is significanlty lower compared to firing rate in wave theory.

lightarrow said:
1. Your description of the "Quantum interpretation" is wrong, as others have said, because "a photon that travels through either slit A or slit B" is exactly a photon of which you have the "which way" information, and that means no interference pattern. Example: you detect the photon near one of the slits putting there a little detector; clearly this is not the two slits experiment we are talking about.
This is just me expressing myself poorly as usual. By saying "either slit A or slit B", I'm not implying that we have any which-way information, just that - after the collapse of the wave function - the quanta travels through either slit A or B. But we don't know which. But we don't need to consider any which-way information to determine the two count rates.

lightarrow said:
2. About your wave interpretation. As I've written in my previos post, if you are certain that the particle splits in two and passes through both slits, nothing changes, it's as if you had two photons passing simultaneously through both slits; it could seem strange, but you have the "which way" information the same, and so no interference pattern. Think to the fact that to say "the particles splits in two and both passes through the slits" means that you have that specific information, and to have it you should put two little detectors near every slit and detect the two particles (the two "half photon") near the slits; so, no interference pattern. Unless you invoke De Broglie Bohm interpretation, the only possibility is that no particles passes through one or the other or both slits. (In my opinion).
I have read a lot about which-way tests but haven't seen one that wouldn't alter the conditions for interference in the first place. But all this is irrelevant, we don't need any which-way information to determine count rates. We don't need the count rate of slit A and B seperately, only the total number that was detected and the total number that was fired.
 
  • #25
Hydr0matic said:
2. I'm not satisfied with a reference to this experiment because it has other components that we don't share the same interpretation about. So in order for me to argue the detection rate issue, I would first have to argue the beam splitting issue for example. And I would prefer not to because it's not really relevant to the double slit experiment, and the key issue is much simpler. All you need is a photon gun, a double slit, a detector and two values - one count rate when there is no double slit, and one when there is.

Why is the beam splitter not identical to the double slit? After all, it provides the superposition of path the SAME way as the 2-slit experiment.

You still haven't mentioned how you are going to reconcile with the FACT that you get single DOTS on a screen when low-intensity light sources are used.

1. So, if I'm wrong, what would QM predict with regards to the detection rate vs firing rate?

I believe both Cthugha and muppet had clearly described this already. I don't quite understand what more you want.

Edit: BTW, I've pointed out Marcella's paper several time already. It might be useful to review the QM derivation of the single, double, and multiple slit formalism without invoking ANY classical wave picture, and yet, one gets exactly the expected results that we know and love. So this is a reference one might want to keep:

T.V. Marcella Eur. J. Phys. v.23, p.615 (2002).

The preprint can be obtained here:

http://arxiv.org/ftp/quant-ph/papers/0703/0703126.pdf

Zz.
 
Last edited:
  • #26
Hydr0matic said:
Not true. This is what QM predicts, not wave theory. In wave theory, there are just as many photons hitting the dark fringes as there is hitting the bright fringes, only they're not setting off the detector because in these exact positions, they are interering destructively. Therefore, the detection count is significanlty lower compared to firing rate in wave theory.

No, this is exactly the wrong point here. I do not know of any theory, which would predict such strange behaviour, especially not wave theory. You just neglect constructive interference. Consider the case of an ideal detector with 100% quantum efficiency, which means that any photon will be detected. While your understanding of destructive interference would cause photons to go through undetected. Howwever at the places of constructive interference nothing would happen. Any photon would be detected anyway due to the 100% QE, so there are just two conclusions left:

a) The constructive interference will lead to an increase in photon detections, which cancels the decrease of the destructive interference area. Therefore no decrease in the total number of photon counts will occur.

b) There are no effects of constructive interference at all. The number of photons detected at the areas of wannabe constructive interference equals the number of photons detected with no double slit present. This is either a violation of conservation of energy or (if you think that the wave will just travel through the screen and be detected somewhere else - however this might happen) easily disproved by putting a second detector screen after the first one, which should then show bright spots, where there have been areas of destructive interference at the first screen. You can easily build your own double slit at home and show, that the second option is plain wrong.
 
  • #27
ZapperZ said:
Why is the beam splitter not identical to the double slit? After all, it provides the superposition of path the SAME way as the 2-slit experiment.
From a QM point of view there's no difference I'm sure, there's only two outcomes - detection or no detection. But if you consider a wave, the process of beamsplitting is an imperfect one, there's not just a right or left choice. With low-intensity waves resulting intensities may be considerably affected by properties of the beamsplitter itself, microscopic irregularities or other discrepancies. And measuring the splitted waves would not be the same as measuring the actual superposition.

ZapperZ said:
You still haven't mentioned how you are going to reconcile with the FACT that you get single DOTS on a screen when low-intensity light sources are used.
You get dots because there is a limitation to how low-intensity light we actually can detect. What you would consider a photon, is what I would define as the lowest intensity light we're able to detect. I believe there is a non-uniform intensity distribution covering the detectors constantly, but simply too low-intensity to detect. So a detection by the CCD would in this scenario represent a higher-intensity-anomaly in the intensity distribution, and the source is filtered to such an extent that these events are so rare as to be considered "individual photons".

ZapperZ said:
I believe both Cthugha and muppet had clearly described this already. I don't quite understand what more you want.
There must be some data on this. Obviously we're not going to convince each other of either interpretation, so there's no point debating. Surely someone has compared counts with and without the double slit? There's probably people here that can do it at work.

ZapperZ said:
Edit: BTW, I've pointed out Marcella's paper several time already. It might be useful to review the QM derivation of the single, double, and multiple slit formalism without invoking ANY classical wave picture, and yet, one gets exactly the expected results that we know and love. So this is a reference one might want to keep:
I don't see relevance. The probability distribution is not what sets the interpretations apart.
 
  • #28
Cthugha said:
No, this is exactly the wrong point here. I do not know of any theory, which would predict such strange behaviour, especially not wave theory. You just neglect constructive interference.
How do I neglect constructive interference?

Cthugha said:
Consider the case of an ideal detector with 100% quantum efficiency, which means that any photon will be detected. While your understanding of destructive interference would cause photons to go through undetected. Howwever at the places of constructive interference nothing would happen.
define "nothing". And I'm not saying the photons will "go through" the detector. I'm not sure what happens with them, maybe they're reflected and scattered?

Cthugha said:
a) The constructive interference will lead to an increase in photon detections, which cancels the decrease of the destructive interference area. Therefore no decrease in the total number of photon counts will occur.
This is the QM interpretation, yes. Any change will only redistribute the photons, so the total count remains the same.

Cthugha said:
b) There are no effects of constructive interference at all. The number of photons detected at the areas of wannabe constructive interference equals the number of photons detected with no double slit present. This is either a violation of conservation of energy or (if you think that the wave will just travel through the screen and be detected somewhere else - however this might happen) easily disproved by putting a second detector screen after the first one, which should then show bright spots, where there have been areas of destructive interference at the first screen. You can easily build your own double slit at home and show, that the second option is plain wrong.
I don't believe they will go through the detector, no. But you could test it by splitting the detector like I illustrated in my initial setup C:
http://physics.soderholms.com/doubleslit_setup.jpg
 
Last edited by a moderator:
  • #29
ZapperZ said:
You still haven't mentioned how you are going to reconcile with the FACT that you get single DOTS on a screen when low-intensity light sources are used.

Zz.


I would like to know more about this. could it be that the energy isn't absorbed directly by the atoms but rather by the field produced by the oscillating charges within the atom which are in turn produced by the absorption of light energy? if the process is unstable then one atom could end up with all the energy thereby starving the surrounding atoms.
 
  • #30
granpa said:
I would like to know more about this. could it be that the energy isn't absorbed directly by the atoms but rather by the field produced by the oscillating charges within the atom which are in turn produced by the absorption of light energy? if the process is unstable then one atom could end up with all the energy thereby starving the surrounding atoms.

After all, the emission of light is accomplished by the surrounding field. why not the absorption of light?
 
  • #31
Hydr0matic said:
How do I neglect constructive interference?

As I said before: imagine an ideal detector behind a double slit. According to your theory, there won't be any redistributions of photons from areas of destructive interference towards areas of constructive interference. So in the areas of (pseudo) constructive interference there will be at most the number of counts, which would be there without the double slit, but no additional counts as the detector is already ideal and there is no redistribution. So in your theory constructive interference is just not there and does not have any effect.

Hydr0matic said:
define "nothing". And I'm not saying the photons will "go through" the detector. I'm not sure what happens with them, maybe they're reflected and scattered?

What are you saying then? Reflected by what? Which wave theory describes such strange behaviour? Not even common classical wave theory does. What should happen to these photons?

Hydr0matic said:
There must be some data on this. Obviously we're not going to convince each other of either interpretation, so there's no point debating. Surely someone has compared counts with and without the double slit? There's probably people here that can do it at work.

What is wrong with the Thorn paper?
However, you can try your idea at home. Building a double slit should not be much of a problem and some low cost photodiodes or a cheap CCD should not be too expensive as well.
 
  • #32
Hydr0matic said:
From a QM point of view there's no difference I'm sure, there's only two outcomes - detection or no detection. But if you consider a wave, the process of beamsplitting is an imperfect one, there's not just a right or left choice. With low-intensity waves resulting intensities may be considerably affected by properties of the beamsplitter itself, microscopic irregularities or other discrepancies. And measuring the splitted waves would not be the same as measuring the actual superposition.

If you are so worried about that, then you need to worry about your "light detector" as well! What instrument do you think you will need to "count" these photons to verify your claim? What principle do these photodetectors use?

You get dots because there is a limitation to how low-intensity light we actually can detect. What you would consider a photon, is what I would define as the lowest intensity light we're able to detect. I believe there is a non-uniform intensity distribution covering the detectors constantly, but simply too low-intensity to detect. So a detection by the CCD would in this scenario represent a higher-intensity-anomaly in the intensity distribution, and the source is filtered to such an extent that these events are so rare as to be considered "individual photons".

That would make no sense that these non-uniformity would somehow conspire to produce JUST coincidentally a pattern that eventually matches the double slit interference pattern. I don't buy that, and this is true, then I could use the SAME argument against your setup, which makes any kind of detection of any kind suspect. I would bring out the CCD data from my photoemission experiment as proof that your assertion is wrong, and that each of these "dots" DO correspond to the location where such a photon.

Zz.
 
  • #33
ZapperZ said:
If you are so worried about that, then you need to worry about your "light detector" as well! What instrument do you think you will need to "count" these photons to verify your claim? What principle do these photodetectors use?
I do worry. Both source and detector are imperfect, which is why I'd prefer no other manipulation between them.

ZapperZ said:
That would make no sense that these non-uniformity would somehow conspire to produce JUST coincidentally a pattern that eventually matches the double slit interference pattern. I don't buy that, and this is true, then I could use the SAME argument against your setup, which makes any kind of detection of any kind suspect. I would bring out the CCD data from my photoemission experiment as proof that your assertion is wrong, and that each of these "dots" DO correspond to the location where such a photon.
You asked me how a singe dot could be produced, not a single dot in an interference pattern. Ofcourse, if you add a double slit the principle would be the same, only with interference.
 
  • #34
Hydr0matic said:
You asked me how a singe dot could be produced, not a single dot in an interference pattern. Ofcourse, if you add a double slit the principle would be the same, only with interference.

Er.. no, I asked about a single dot on the detector produced by sending light of extremely low intensity through the double slit. So the double slit IS there.

BTW, based on Marcella's paper, where does it confirm your claim of what QM would produce using your scenario?

Zz.
 
  • #35
ZapperZ said:
You still haven't mentioned how you are going to reconcile with the FACT that you get single DOTS on a screen when low-intensity light sources are used.
Ok, sorry. I should have assumed you meant through a double slit.
 

Similar threads

Replies
32
Views
2K
  • Quantum Physics
2
Replies
36
Views
1K
  • Quantum Physics
Replies
18
Views
1K
  • Quantum Physics
Replies
14
Views
1K
Replies
5
Views
773
Replies
3
Views
773
  • Quantum Physics
Replies
9
Views
767
Replies
19
Views
946
Replies
28
Views
534
Replies
18
Views
2K
Back
Top